1
$\begingroup$

Determine up to similarity all $3 \times 3$ complex matrices $A$ such that $A^4 + 2A^3 + A^2 = 0$ and $A^2 + A \neq 0$. Give the characteristic and minimal polynomial of each matrix.

I'm not quite sure how to approach this problem. First, it's clear that the minimal polynomial of any matrix which satisfies these conditions must divide, \begin{align*} p(x) = x^4 + 2x^3 + x^2 = 0 \end{align*}

and one can factor this into $x^2(x+1)(x+1)$. Also, I am aware that similar matrices share the same characteristic/minimal polynomials. There may also, perhaps, be a connection to Jordan Canonical Form. What I am not sure about is, how can we find the characteristic polynomial, how exactly does the condition $A^2 + A \neq 0$ factor into this, and how do I know when I've found all of the matrices.

$\endgroup$
12
  • 1
    $\begingroup$ I don't have the solution, but since $A(A+1)\ne0$, the minimal polynomial cannot be a divisor of $x(x+1)$. There are now very few possible options for the minimal polynomial. $\endgroup$
    – Milten
    Commented Jan 5 at 7:55
  • 1
    $\begingroup$ And yes, from that point I would probably list the possible cases of Jordan normal forms. $\endgroup$
    – Milten
    Commented Jan 5 at 7:59
  • $\begingroup$ Ah I see, so the only valid minimal polynomials would be $x^2$, $x^2(x+1)$,$x(x+1)^2$, or $x^2(x+1)^2$. How does the characteristic polynomial come into play here? $\endgroup$ Commented Jan 5 at 8:18
  • $\begingroup$ You can find the characteristic polynomial from the Jordan normal form. You don't need it to list the matrices. $\endgroup$
    – Milten
    Commented Jan 5 at 8:27
  • $\begingroup$ Are you familiar with Jordan's canonical form? If you are, then suppose $J$ is the Jordan's canonical form of $A$, with $J=PAP^{-1}$. Then we have \begin{align*} &A^4 + 2A^3 + A^2 = 0 &\\ \iff & P(A^4 + 2A^3 + A^2 )P^{-1}= 0 & (\text{since $P0P^{-1}=0$})\\ \iff & J^4 + 2J^3 + J^2 = 0 & (\text{since $PA^nP^{-1}=PAP^{-1}PAP^{-1}\cdots PAP^{-1}=J^n$}) \end{align*} The same argument gives $J^2+J\ne 0$. $\endgroup$
    – Asigan
    Commented Jan 5 at 8:54

1 Answer 1

1
$\begingroup$

To make the answer complete I first repeat some of my comment. Suppose $J$ is the Jordan's canonical form of $A$, with $J=PAP^{-1}$. Then we have \begin{align*} &A^4 + 2A^3 + A^2 = 0 &\\ \iff & P(A^4 + 2A^3 + A^2 )P^{-1}= 0 & (\text{since $P0P^{-1}=0$})\\ \iff & J^4 + 2J^3 + J^2 = 0 & (\text{since $PA^nP^{-1}=PAP^{-1}PAP^{-1}\cdots PAP^{-1}=J^n$}) \end{align*} The same argument gives $J^2+J\ne 0$. So it suffices to determine all the Jordan-canonical form $J$ s.t. $J^4 + 2J^3 + J^2 = 0$, $J^2+J\ne 0$. There are three cases according to the sizes and numbers Jordan blocks. We are going to check them case by case.

For convinience denote $f(x)=x^4+2x^3+x^2$, $ g(x)=x^2+x $. Note that $f(x_0)=0$ $\implies$ $x_0\in\{0,-1\}$ $\implies$ $g(x_0)=0$.

Case 1: $J$ has Jordan blocks "$1+1+1$". (That is it has three Jordan blocks, each with size one.)

Then $J=\mathtt{diag}\{\lambda,\mu, \omega\}$, where $\lambda,\mu, \omega$ need not be distinct. Since $J^4 + 2J^3 + J^2 = 0$, we have $f(\lambda)=f(\mu)=f(\omega)=0$. Therefore $\lambda,\mu, \omega\in\{0,-1\}$. But this implies $\lambda,\mu, \omega$ are zeros of $g(x)$. Hence $J^2+J=\mathtt{diag}\{g(\lambda),g(\mu), g(\omega)\}=0$, a contradiction. Therefore Case 1 is impossible.

Case 2: $J$ has Jordan blocks "$2+1$".

Then \begin{equation*} J=\begin{pmatrix} \lambda & 1 & \\ & \lambda & \\ & & \mu \end{pmatrix}, \end{equation*} where $\lambda,\mu$ need not be distinct. A direct computation gives \begin{equation*} J^4 + 2J^3 + J^2 =\begin{pmatrix} f(\lambda) & 4\lambda^3+6\lambda^3+2\lambda^2 & \\ & f(\lambda) & \\ & & f(\mu) \end{pmatrix}. \end{equation*} So we must have $f(\lambda)=f(\mu)=4\lambda^3+6\lambda^3+2\lambda^2=0$. This gives $\lambda,\mu \in\{-1,0\}$.

Another direct computation gives \begin{equation*} J^2+J=\begin{pmatrix} \lambda^2+\lambda & 2\lambda+1 & \\ & \lambda^2+\lambda & \\ & & \mu^2+\mu \end{pmatrix}. \end{equation*} When $\lambda\in\{0,-1\}$, $2\lambda+1\ne 0$. Therefore $J^2+J\ne 0$ whenever $\lambda\in\{0,-1\}$.

Hence case 2 gives four solutions: \begin{equation*} J=\begin{pmatrix} \lambda & 1 & \\ & \lambda & \\ & & \mu \end{pmatrix}, \end{equation*} where $\lambda,\mu\in\{0,-1\}$.

Case three: $J$ has Jordan blocks "$3$". Then \begin{equation*} J=\begin{pmatrix} \lambda & 1 & \\ & \lambda & 1 \\ & & \lambda \end{pmatrix} . \end{equation*} A direct proof gives \begin{equation*} J^4 + 2J^3 + J^2=\begin{pmatrix} f(\lambda) & \text{the value here is not important} & 4\lambda^2+3\lambda+1 \\ & f(\lambda) & \text{the value here is not important} \\ & & f(\lambda) \end{pmatrix} . \end{equation*} Since $f(\lambda)$ and $ 4\lambda^2+3\lambda+1 $ can not be zero at the same time, we conclude Case three is impossible.

To sum up there are (up to similarity) four choices for $J$ (and hence for $A$): \begin{equation*} J=\begin{pmatrix} \lambda & 1 & \\ & \lambda & \\ & & \mu \end{pmatrix}, \end{equation*} where $\lambda,\mu\in\{0,-1\}$.

$\endgroup$
1
  • $\begingroup$ wow thanks a lot for the thorough answer. $\endgroup$ Commented Jan 13 at 16:42

You must log in to answer this question.

Not the answer you're looking for? Browse other questions tagged .